LSAT and Law School Admissions Forum

Get expert LSAT preparation and law school admissions advice from PowerScore Test Preparation.

User avatar
 Dave Killoran
PowerScore Staff
  • PowerScore Staff
  • Posts: 5852
  • Joined: Mar 25, 2011
|
#80193
Complete Question Explanation
(The complete setup for this game can be found here: lsat/viewtopic.php?t=885)

The correct answer choice is (A)

This question is similar to the preceding question, with the addition that light 5 is on. When light 5 is on, then Template #1 or #2 can apply, with the specification that in Template #1 light 4 would be off.

Answer choice (A) is the correct answer. Under Template #2 it is possible for lights 1 and 6 to be off.

Answer choice (B) is incorrect because from the third rule, if light 1 is on then light 7 is off.

Answer choice (C) is incorrect because light 4 is always off when light 5 is on.

Answer choice (D) is incorrect because if light 2 is off, then light 6 is on.

Answer choice (E) is incorrect because this would cause four consecutively numbered lights to be on (5, 6, 7, 8), a multiple violation of the first rule.

Get the most out of your LSAT Prep Plus subscription.

Analyze and track your performance with our Testing and Analytics Package.